An application of Banach-Steinhaus theorem

Functional Analysis
Post Reply
r9m
Posts: 59
Joined: Thu Dec 10, 2015 1:58 pm
Location: India
Contact:

An application of Banach-Steinhaus theorem

#1

Post by r9m »

Suppose $1 < p < \infty$ and $p,q$ are conjugate indices, i.e., $\displaystyle \frac{1}{p} + \frac{1}{q} = 1$. If $(\mathbb{R},\mu)$ be the Lebesgue measure. If the following properties hold:

(i) $\displaystyle g \in L^{q}_{\text{loc}}(\mu)$.

(ii) $\displaystyle \int_{\mathbb{R}} |fg|\,d\mu < \infty$, for all $f \in L^{p}(\mu)$.

Then apply Banach-Steinhaus (Uniform Boundedness principle) to show that $g \in L^{q}(\mu)$.
Post Reply

Create an account or sign in to join the discussion

You need to be a member in order to post a reply

Create an account

Not a member? register to join our community
Members can start their own topics & subscribe to topics
It’s free and only takes a minute

Register

Sign in

Who is online

Users browsing this forum: No registered users and 7 guests